LSAT and Law School Admissions Forum

Get expert LSAT preparation and law school admissions advice from PowerScore Test Preparation.

 Administrator
PowerScore Staff
  • PowerScore Staff
  • Posts: 8917
  • Joined: Feb 02, 2011
|
#71198
Please post your questions below! Thank you!
User avatar
 appletree
  • Posts: 13
  • Joined: Feb 11, 2021
|
#87646
Hello,
I picked answer C because all of the other answer choices were very unsupported. However, I am finding a hard time finding good justification for answer C.
The only evidence I could find was at the end of the second paragraph with the sentence "experience shows that harmful pollutants... rarely attempted to remedy this situation."
I can see this supports the idea that nations are not really held accountable because they keep doing the same bad stuff over and over, but saying they are rarely LEGALLY held accountable based on this seems a bit too strong?
Could someone please clarify? Is there some additional evidence I am missing?
Thank you!
User avatar
 Ryan Twomey
PowerScore Staff
  • PowerScore Staff
  • Posts: 141
  • Joined: Mar 04, 2021
|
#87697
Hey Appletree,

So this is a great question. And you may already know this, but I always like to reiterate this in the more difficult must be true (most strongly supported) questions in reading comp: they are asking which answer choice is the most strongly supported out of the five, not which answer is definitively true.

But I do think the sentence you point outsupports answer choice C. I will post it here for the other students who view this in the future: "Although systematic empirical studies are lacking, experience shows that harmful pollutants constantly cross most international borders, and that nations have only rarely attempted to remedy this situation."

This was considered a principle by the author in previous sentences, and it has been "rarely" even been attempted to be remedied. The first sentence in paragraph 3 also supports this answer choice.

I don't think this answer choice is by any means 100% certainly true, but based on the passage, I'd give it an 80-90% chance of being true, which is higher than the remaining 4 answer choices, making this answer choice the most strongly supported.

I hope this helps, and I wish you all of the luck in your studies.

Best,
Ryan

Get the most out of your LSAT Prep Plus subscription.

Analyze and track your performance with our Testing and Analytics Package.